146 lines
2.8 KiB
TeX
146 lines
2.8 KiB
TeX
|
\section{Defining $e$}
|
||
|
|
||
|
\problem{}
|
||
|
Recall and prove the binomial theorem.
|
||
|
|
||
|
\begin{solution}
|
||
|
The binomial theorem:
|
||
|
$$(a + b)^n = \sum_{i=0}^{n} \binom{n}{i}a^ib^{n-i}$$
|
||
|
|
||
|
We usually prove this by induction.
|
||
|
\end{solution}
|
||
|
|
||
|
\vfill
|
||
|
|
||
|
|
||
|
|
||
|
|
||
|
\problem{}<e_n>
|
||
|
Prove the following:
|
||
|
|
||
|
$$ \bigg(1 + \frac{1}{n} \bigg)^n < 3 - \frac{1}{n}\ \text{for all } n = 3, 4, 5, ...$$
|
||
|
|
||
|
This proves that that the sequence $e_n = (1 + \frac{1}{n})^n$, $n = 1, 2, ...$ is bounded from above, \par
|
||
|
since $e_n < 3\ \forall n \in \mathbb{N}$.
|
||
|
|
||
|
\begin{solution}
|
||
|
$$
|
||
|
\bigg(1 + \frac{1}{n}\bigg)^n =
|
||
|
\sum_{k=0}^n \binom{n}{k}\frac{1}{n^i} =
|
||
|
2 + \sum_{k=2}^n \binom{n}{k}\frac{1}{n^i} =
|
||
|
2 + \sum_{k=2}^n \frac{1}{k!}\frac{(n)(n-1)...(n-k+1)}{n^k} <
|
||
|
2 + \sum_{k=2}^n \frac{1}{k!}
|
||
|
$$
|
||
|
|
||
|
$$
|
||
|
2 + \sum_{k=2}^n \frac{1}{k!} <
|
||
|
2 + \sum_{k=2}^n \frac{1}{k(k-1)} =
|
||
|
2 + 1 - \frac{1}{n} = 3 - \frac{1}{n}
|
||
|
$$
|
||
|
\end{solution}
|
||
|
|
||
|
\vfill
|
||
|
\pagebreak
|
||
|
|
||
|
|
||
|
|
||
|
\theorem{Bernoulli's inequality}<bernoulli>
|
||
|
$(x + 1)^n \geq 1 + nx$ for $x \geq -1$ and $n \in \mathbb{N}$
|
||
|
|
||
|
\problem{}
|
||
|
Use induction to prove \ref{bernoulli}
|
||
|
|
||
|
\vfill
|
||
|
|
||
|
|
||
|
|
||
|
\problem{}<e_n_inc>
|
||
|
Use \ref{bernoulli} to prove that the $e_n$ defined in \ref{e_n} is monotonically increasing.
|
||
|
|
||
|
\begin{solution}
|
||
|
We want to show that the following is true:
|
||
|
$$
|
||
|
\bigg(
|
||
|
1 + \frac{1}{n + 1}
|
||
|
\bigg)^{n+1}
|
||
|
>
|
||
|
\bigg(
|
||
|
1 + \frac{1}{n}
|
||
|
\bigg)^n
|
||
|
$$
|
||
|
|
||
|
This inequality is equivalent to
|
||
|
$$
|
||
|
\Bigg(
|
||
|
\frac{
|
||
|
1 + \frac{1}{n+1}
|
||
|
}{
|
||
|
1 + \frac{1}{n}
|
||
|
}
|
||
|
\Bigg)^{n+1}
|
||
|
>
|
||
|
\bigg(
|
||
|
1 + \frac{1}{n}
|
||
|
\bigg)^{-1}
|
||
|
= \frac{1}{1 + \frac{1}{n}}
|
||
|
= \frac{n}{n+1}
|
||
|
= 1 - \frac{1}{n+1}
|
||
|
$$
|
||
|
|
||
|
Also,
|
||
|
$$
|
||
|
\frac{
|
||
|
1 + \frac{1}{n+1}
|
||
|
} {
|
||
|
1 + \frac{1}{n}
|
||
|
}
|
||
|
= 1 - \frac{1}{(n + 1)^2}
|
||
|
$$
|
||
|
|
||
|
\ref{bernoulli} tells us that
|
||
|
$$
|
||
|
\bigg(
|
||
|
1 - \frac{1}{(n+1)^2}
|
||
|
\bigg) ^ {n+1}
|
||
|
= 1 - \frac{n+1}{(n+1)^2}
|
||
|
= 1 - \frac{1}{n+1}
|
||
|
$$
|
||
|
|
||
|
Since this is equivalent to our original inequality, we are done.
|
||
|
|
||
|
\end{solution}
|
||
|
|
||
|
\vfill
|
||
|
\pagebreak
|
||
|
|
||
|
|
||
|
|
||
|
\definition{}
|
||
|
\ref{e_n}, \ref{e_n_inc}, and \ref{limexists} tell us that $e_n$ has a limit. \par
|
||
|
Let us define $e$:
|
||
|
$$e = \lim_{n\to\infty}{e_n} = \lim_{n\to\infty}{\bigg(1 + \frac{1}{n} \bigg)^n}$$
|
||
|
|
||
|
\vfill
|
||
|
|
||
|
|
||
|
\problem{}
|
||
|
Derive the formula for $V(t)$ if the annual rate $r$ is compounded continuously.
|
||
|
|
||
|
\begin{solution}
|
||
|
For a rate $r$ compounded $n$ times per year, we have $V(t) = P(1 + \frac{r}{n})^{nt}$.
|
||
|
|
||
|
$\lim_{n\to\infty}{(V(t))} = \lim_{n\to\infty}{P(1 + \frac{r}{n})^{nt}}$
|
||
|
|
||
|
Let $a = \frac{n}{r}$ \par
|
||
|
|
||
|
$\lim_{n\to\infty}{a} = \infty$, so $\lim_{n\to\infty}{V(t)} = \lim_{a\to\infty}{V(t)}$ \par
|
||
|
|
||
|
Substituting $a$ for $n$, we get \par
|
||
|
$P(1 + \frac{r}{n})^{nt} = P(1 + \frac{1}{a})^{art}$ \par
|
||
|
|
||
|
And finally, we can evaluate \par
|
||
|
$\lim_{a\to\infty}{P (1 + \frac{1}{a})^{art}} = Pe^{rt}$
|
||
|
\end{solution}
|
||
|
|
||
|
\vfill
|
||
|
\pagebreak
|